Burgers' equation $u_y + uu_x = 0$ with $u(x,0)=-x$












2












$begingroup$


I couldn't solve this problem, can you help me please?



The Burgers' equation



$$ u_y + uu_x = 0 $$ $ - infty < x < infty $ , $ y > 0 $ , $ u(x,0)=f(x) $



My question;



is there any solution $ f(x)= -x $ for all $ y>0 $ , if not, why?



Thank you for your help.










share|cite|improve this question











$endgroup$












  • $begingroup$
    You can use the method of characteristics to solve it.
    $endgroup$
    – Mhenni Benghorbal
    Jul 29 '13 at 12:11










  • $begingroup$
    Hi Dear, I tried but it doesn't work.
    $endgroup$
    – John
    Jul 29 '13 at 12:34










  • $begingroup$
    See here.
    $endgroup$
    – Mhenni Benghorbal
    Jul 29 '13 at 13:18
















2












$begingroup$


I couldn't solve this problem, can you help me please?



The Burgers' equation



$$ u_y + uu_x = 0 $$ $ - infty < x < infty $ , $ y > 0 $ , $ u(x,0)=f(x) $



My question;



is there any solution $ f(x)= -x $ for all $ y>0 $ , if not, why?



Thank you for your help.










share|cite|improve this question











$endgroup$












  • $begingroup$
    You can use the method of characteristics to solve it.
    $endgroup$
    – Mhenni Benghorbal
    Jul 29 '13 at 12:11










  • $begingroup$
    Hi Dear, I tried but it doesn't work.
    $endgroup$
    – John
    Jul 29 '13 at 12:34










  • $begingroup$
    See here.
    $endgroup$
    – Mhenni Benghorbal
    Jul 29 '13 at 13:18














2












2








2





$begingroup$


I couldn't solve this problem, can you help me please?



The Burgers' equation



$$ u_y + uu_x = 0 $$ $ - infty < x < infty $ , $ y > 0 $ , $ u(x,0)=f(x) $



My question;



is there any solution $ f(x)= -x $ for all $ y>0 $ , if not, why?



Thank you for your help.










share|cite|improve this question











$endgroup$




I couldn't solve this problem, can you help me please?



The Burgers' equation



$$ u_y + uu_x = 0 $$ $ - infty < x < infty $ , $ y > 0 $ , $ u(x,0)=f(x) $



My question;



is there any solution $ f(x)= -x $ for all $ y>0 $ , if not, why?



Thank you for your help.







pde hyperbolic-equations






share|cite|improve this question















share|cite|improve this question













share|cite|improve this question




share|cite|improve this question








edited Dec 7 '18 at 13:20









Harry49

6,22331132




6,22331132










asked Jul 29 '13 at 11:29









JohnJohn

132




132












  • $begingroup$
    You can use the method of characteristics to solve it.
    $endgroup$
    – Mhenni Benghorbal
    Jul 29 '13 at 12:11










  • $begingroup$
    Hi Dear, I tried but it doesn't work.
    $endgroup$
    – John
    Jul 29 '13 at 12:34










  • $begingroup$
    See here.
    $endgroup$
    – Mhenni Benghorbal
    Jul 29 '13 at 13:18


















  • $begingroup$
    You can use the method of characteristics to solve it.
    $endgroup$
    – Mhenni Benghorbal
    Jul 29 '13 at 12:11










  • $begingroup$
    Hi Dear, I tried but it doesn't work.
    $endgroup$
    – John
    Jul 29 '13 at 12:34










  • $begingroup$
    See here.
    $endgroup$
    – Mhenni Benghorbal
    Jul 29 '13 at 13:18
















$begingroup$
You can use the method of characteristics to solve it.
$endgroup$
– Mhenni Benghorbal
Jul 29 '13 at 12:11




$begingroup$
You can use the method of characteristics to solve it.
$endgroup$
– Mhenni Benghorbal
Jul 29 '13 at 12:11












$begingroup$
Hi Dear, I tried but it doesn't work.
$endgroup$
– John
Jul 29 '13 at 12:34




$begingroup$
Hi Dear, I tried but it doesn't work.
$endgroup$
– John
Jul 29 '13 at 12:34












$begingroup$
See here.
$endgroup$
– Mhenni Benghorbal
Jul 29 '13 at 13:18




$begingroup$
See here.
$endgroup$
– Mhenni Benghorbal
Jul 29 '13 at 13:18










2 Answers
2






active

oldest

votes


















4












$begingroup$

I rewrite equation to common view



$$frac{partial u}{partial t}+ufrac{partial u}{partial x}=0$$
There is a solution with each initial value, but this solution exists only for $tle t_{r}$, where $t_{r}$ is a point where the phenomenon of "rollover" occurs. This phenomenon occurs if some characteristics of this equations intersect. Here is a picture of characteristics:
enter image description here



It shows that characteristics of this equations intersect on the moment $t=1$



Before the moment $t=1$ the solution is:



$$u(x,t)=f(x-ut),$$ where $$f(x)=-x $$



After the point $t=1$ the solution does not exist. There you should change your equation to conservation law:



$$frac{partial u}{partial t}+frac{partial}{partial x}left(frac{u^2}{2}right)=0$$



and define the order of this law.






share|cite|improve this answer











$endgroup$













  • $begingroup$
    Here is a mistake. Wait for 1 minute..
    $endgroup$
    – cool
    Jul 29 '13 at 14:01










  • $begingroup$
    Please,look at the answer,@John
    $endgroup$
    – cool
    Jul 29 '13 at 14:11










  • $begingroup$
    Yes,it's correct. This picture shows that there is no solution for $t>1$, but previous showed that there was, that was incorrect.
    $endgroup$
    – cool
    Jul 29 '13 at 14:19










  • $begingroup$
    @John "There you should change your equation to conservation law"... I would rather say that the classical solution breaks down at $t=1$ (it then equals simultaneously all values from $-infty$ to $+infty$). And there is no weak solution to start after this blow-up happened.
    $endgroup$
    – Harry49
    Dec 5 '18 at 17:54





















0












$begingroup$

Follow the method in http://en.wikipedia.org/wiki/Method_of_characteristics#Example:



$dfrac{dy}{dt}=1$ , letting $y(0)=0$ , we have $y=t$



$dfrac{du}{dt}=0$ , letting $u(0)=u_0$ , we have $u=u_0$



$dfrac{dx}{dt}=u=u_0$ , letting $x(0)=f(u_0)$ , we have $x=u_0t+f(u_0)=uy+f(u)$ , i.e. $u=F(x-uy)$



$u(x,0)=-x$ :



$F(x)=-x$



$therefore u=-(x-uy)=-x+uy$



Since $u(x,y)$ has perfect explicit form $u(x,y)=dfrac{x}{y-1}$ , of course it has solution for all $y>0$ .






share|cite|improve this answer









$endgroup$









  • 1




    $begingroup$
    I consider this solution be wrong (partially). The picture higher shows the real situation with characteristics -- they intersect at $y=1$, beginning from that point solution does not exist.
    $endgroup$
    – cool
    Jul 30 '13 at 13:09











Your Answer





StackExchange.ifUsing("editor", function () {
return StackExchange.using("mathjaxEditing", function () {
StackExchange.MarkdownEditor.creationCallbacks.add(function (editor, postfix) {
StackExchange.mathjaxEditing.prepareWmdForMathJax(editor, postfix, [["$", "$"], ["\\(","\\)"]]);
});
});
}, "mathjax-editing");

StackExchange.ready(function() {
var channelOptions = {
tags: "".split(" "),
id: "69"
};
initTagRenderer("".split(" "), "".split(" "), channelOptions);

StackExchange.using("externalEditor", function() {
// Have to fire editor after snippets, if snippets enabled
if (StackExchange.settings.snippets.snippetsEnabled) {
StackExchange.using("snippets", function() {
createEditor();
});
}
else {
createEditor();
}
});

function createEditor() {
StackExchange.prepareEditor({
heartbeatType: 'answer',
autoActivateHeartbeat: false,
convertImagesToLinks: true,
noModals: true,
showLowRepImageUploadWarning: true,
reputationToPostImages: 10,
bindNavPrevention: true,
postfix: "",
imageUploader: {
brandingHtml: "Powered by u003ca class="icon-imgur-white" href="https://imgur.com/"u003eu003c/au003e",
contentPolicyHtml: "User contributions licensed under u003ca href="https://creativecommons.org/licenses/by-sa/3.0/"u003ecc by-sa 3.0 with attribution requiredu003c/au003e u003ca href="https://stackoverflow.com/legal/content-policy"u003e(content policy)u003c/au003e",
allowUrls: true
},
noCode: true, onDemand: true,
discardSelector: ".discard-answer"
,immediatelyShowMarkdownHelp:true
});


}
});














draft saved

draft discarded


















StackExchange.ready(
function () {
StackExchange.openid.initPostLogin('.new-post-login', 'https%3a%2f%2fmath.stackexchange.com%2fquestions%2f454776%2fburgers-equation-u-y-uu-x-0-with-ux-0-x%23new-answer', 'question_page');
}
);

Post as a guest















Required, but never shown

























2 Answers
2






active

oldest

votes








2 Answers
2






active

oldest

votes









active

oldest

votes






active

oldest

votes









4












$begingroup$

I rewrite equation to common view



$$frac{partial u}{partial t}+ufrac{partial u}{partial x}=0$$
There is a solution with each initial value, but this solution exists only for $tle t_{r}$, where $t_{r}$ is a point where the phenomenon of "rollover" occurs. This phenomenon occurs if some characteristics of this equations intersect. Here is a picture of characteristics:
enter image description here



It shows that characteristics of this equations intersect on the moment $t=1$



Before the moment $t=1$ the solution is:



$$u(x,t)=f(x-ut),$$ where $$f(x)=-x $$



After the point $t=1$ the solution does not exist. There you should change your equation to conservation law:



$$frac{partial u}{partial t}+frac{partial}{partial x}left(frac{u^2}{2}right)=0$$



and define the order of this law.






share|cite|improve this answer











$endgroup$













  • $begingroup$
    Here is a mistake. Wait for 1 minute..
    $endgroup$
    – cool
    Jul 29 '13 at 14:01










  • $begingroup$
    Please,look at the answer,@John
    $endgroup$
    – cool
    Jul 29 '13 at 14:11










  • $begingroup$
    Yes,it's correct. This picture shows that there is no solution for $t>1$, but previous showed that there was, that was incorrect.
    $endgroup$
    – cool
    Jul 29 '13 at 14:19










  • $begingroup$
    @John "There you should change your equation to conservation law"... I would rather say that the classical solution breaks down at $t=1$ (it then equals simultaneously all values from $-infty$ to $+infty$). And there is no weak solution to start after this blow-up happened.
    $endgroup$
    – Harry49
    Dec 5 '18 at 17:54


















4












$begingroup$

I rewrite equation to common view



$$frac{partial u}{partial t}+ufrac{partial u}{partial x}=0$$
There is a solution with each initial value, but this solution exists only for $tle t_{r}$, where $t_{r}$ is a point where the phenomenon of "rollover" occurs. This phenomenon occurs if some characteristics of this equations intersect. Here is a picture of characteristics:
enter image description here



It shows that characteristics of this equations intersect on the moment $t=1$



Before the moment $t=1$ the solution is:



$$u(x,t)=f(x-ut),$$ where $$f(x)=-x $$



After the point $t=1$ the solution does not exist. There you should change your equation to conservation law:



$$frac{partial u}{partial t}+frac{partial}{partial x}left(frac{u^2}{2}right)=0$$



and define the order of this law.






share|cite|improve this answer











$endgroup$













  • $begingroup$
    Here is a mistake. Wait for 1 minute..
    $endgroup$
    – cool
    Jul 29 '13 at 14:01










  • $begingroup$
    Please,look at the answer,@John
    $endgroup$
    – cool
    Jul 29 '13 at 14:11










  • $begingroup$
    Yes,it's correct. This picture shows that there is no solution for $t>1$, but previous showed that there was, that was incorrect.
    $endgroup$
    – cool
    Jul 29 '13 at 14:19










  • $begingroup$
    @John "There you should change your equation to conservation law"... I would rather say that the classical solution breaks down at $t=1$ (it then equals simultaneously all values from $-infty$ to $+infty$). And there is no weak solution to start after this blow-up happened.
    $endgroup$
    – Harry49
    Dec 5 '18 at 17:54
















4












4








4





$begingroup$

I rewrite equation to common view



$$frac{partial u}{partial t}+ufrac{partial u}{partial x}=0$$
There is a solution with each initial value, but this solution exists only for $tle t_{r}$, where $t_{r}$ is a point where the phenomenon of "rollover" occurs. This phenomenon occurs if some characteristics of this equations intersect. Here is a picture of characteristics:
enter image description here



It shows that characteristics of this equations intersect on the moment $t=1$



Before the moment $t=1$ the solution is:



$$u(x,t)=f(x-ut),$$ where $$f(x)=-x $$



After the point $t=1$ the solution does not exist. There you should change your equation to conservation law:



$$frac{partial u}{partial t}+frac{partial}{partial x}left(frac{u^2}{2}right)=0$$



and define the order of this law.






share|cite|improve this answer











$endgroup$



I rewrite equation to common view



$$frac{partial u}{partial t}+ufrac{partial u}{partial x}=0$$
There is a solution with each initial value, but this solution exists only for $tle t_{r}$, where $t_{r}$ is a point where the phenomenon of "rollover" occurs. This phenomenon occurs if some characteristics of this equations intersect. Here is a picture of characteristics:
enter image description here



It shows that characteristics of this equations intersect on the moment $t=1$



Before the moment $t=1$ the solution is:



$$u(x,t)=f(x-ut),$$ where $$f(x)=-x $$



After the point $t=1$ the solution does not exist. There you should change your equation to conservation law:



$$frac{partial u}{partial t}+frac{partial}{partial x}left(frac{u^2}{2}right)=0$$



and define the order of this law.







share|cite|improve this answer














share|cite|improve this answer



share|cite|improve this answer








edited Jul 29 '13 at 14:08

























answered Jul 29 '13 at 12:54









coolcool

901618




901618












  • $begingroup$
    Here is a mistake. Wait for 1 minute..
    $endgroup$
    – cool
    Jul 29 '13 at 14:01










  • $begingroup$
    Please,look at the answer,@John
    $endgroup$
    – cool
    Jul 29 '13 at 14:11










  • $begingroup$
    Yes,it's correct. This picture shows that there is no solution for $t>1$, but previous showed that there was, that was incorrect.
    $endgroup$
    – cool
    Jul 29 '13 at 14:19










  • $begingroup$
    @John "There you should change your equation to conservation law"... I would rather say that the classical solution breaks down at $t=1$ (it then equals simultaneously all values from $-infty$ to $+infty$). And there is no weak solution to start after this blow-up happened.
    $endgroup$
    – Harry49
    Dec 5 '18 at 17:54




















  • $begingroup$
    Here is a mistake. Wait for 1 minute..
    $endgroup$
    – cool
    Jul 29 '13 at 14:01










  • $begingroup$
    Please,look at the answer,@John
    $endgroup$
    – cool
    Jul 29 '13 at 14:11










  • $begingroup$
    Yes,it's correct. This picture shows that there is no solution for $t>1$, but previous showed that there was, that was incorrect.
    $endgroup$
    – cool
    Jul 29 '13 at 14:19










  • $begingroup$
    @John "There you should change your equation to conservation law"... I would rather say that the classical solution breaks down at $t=1$ (it then equals simultaneously all values from $-infty$ to $+infty$). And there is no weak solution to start after this blow-up happened.
    $endgroup$
    – Harry49
    Dec 5 '18 at 17:54


















$begingroup$
Here is a mistake. Wait for 1 minute..
$endgroup$
– cool
Jul 29 '13 at 14:01




$begingroup$
Here is a mistake. Wait for 1 minute..
$endgroup$
– cool
Jul 29 '13 at 14:01












$begingroup$
Please,look at the answer,@John
$endgroup$
– cool
Jul 29 '13 at 14:11




$begingroup$
Please,look at the answer,@John
$endgroup$
– cool
Jul 29 '13 at 14:11












$begingroup$
Yes,it's correct. This picture shows that there is no solution for $t>1$, but previous showed that there was, that was incorrect.
$endgroup$
– cool
Jul 29 '13 at 14:19




$begingroup$
Yes,it's correct. This picture shows that there is no solution for $t>1$, but previous showed that there was, that was incorrect.
$endgroup$
– cool
Jul 29 '13 at 14:19












$begingroup$
@John "There you should change your equation to conservation law"... I would rather say that the classical solution breaks down at $t=1$ (it then equals simultaneously all values from $-infty$ to $+infty$). And there is no weak solution to start after this blow-up happened.
$endgroup$
– Harry49
Dec 5 '18 at 17:54






$begingroup$
@John "There you should change your equation to conservation law"... I would rather say that the classical solution breaks down at $t=1$ (it then equals simultaneously all values from $-infty$ to $+infty$). And there is no weak solution to start after this blow-up happened.
$endgroup$
– Harry49
Dec 5 '18 at 17:54













0












$begingroup$

Follow the method in http://en.wikipedia.org/wiki/Method_of_characteristics#Example:



$dfrac{dy}{dt}=1$ , letting $y(0)=0$ , we have $y=t$



$dfrac{du}{dt}=0$ , letting $u(0)=u_0$ , we have $u=u_0$



$dfrac{dx}{dt}=u=u_0$ , letting $x(0)=f(u_0)$ , we have $x=u_0t+f(u_0)=uy+f(u)$ , i.e. $u=F(x-uy)$



$u(x,0)=-x$ :



$F(x)=-x$



$therefore u=-(x-uy)=-x+uy$



Since $u(x,y)$ has perfect explicit form $u(x,y)=dfrac{x}{y-1}$ , of course it has solution for all $y>0$ .






share|cite|improve this answer









$endgroup$









  • 1




    $begingroup$
    I consider this solution be wrong (partially). The picture higher shows the real situation with characteristics -- they intersect at $y=1$, beginning from that point solution does not exist.
    $endgroup$
    – cool
    Jul 30 '13 at 13:09
















0












$begingroup$

Follow the method in http://en.wikipedia.org/wiki/Method_of_characteristics#Example:



$dfrac{dy}{dt}=1$ , letting $y(0)=0$ , we have $y=t$



$dfrac{du}{dt}=0$ , letting $u(0)=u_0$ , we have $u=u_0$



$dfrac{dx}{dt}=u=u_0$ , letting $x(0)=f(u_0)$ , we have $x=u_0t+f(u_0)=uy+f(u)$ , i.e. $u=F(x-uy)$



$u(x,0)=-x$ :



$F(x)=-x$



$therefore u=-(x-uy)=-x+uy$



Since $u(x,y)$ has perfect explicit form $u(x,y)=dfrac{x}{y-1}$ , of course it has solution for all $y>0$ .






share|cite|improve this answer









$endgroup$









  • 1




    $begingroup$
    I consider this solution be wrong (partially). The picture higher shows the real situation with characteristics -- they intersect at $y=1$, beginning from that point solution does not exist.
    $endgroup$
    – cool
    Jul 30 '13 at 13:09














0












0








0





$begingroup$

Follow the method in http://en.wikipedia.org/wiki/Method_of_characteristics#Example:



$dfrac{dy}{dt}=1$ , letting $y(0)=0$ , we have $y=t$



$dfrac{du}{dt}=0$ , letting $u(0)=u_0$ , we have $u=u_0$



$dfrac{dx}{dt}=u=u_0$ , letting $x(0)=f(u_0)$ , we have $x=u_0t+f(u_0)=uy+f(u)$ , i.e. $u=F(x-uy)$



$u(x,0)=-x$ :



$F(x)=-x$



$therefore u=-(x-uy)=-x+uy$



Since $u(x,y)$ has perfect explicit form $u(x,y)=dfrac{x}{y-1}$ , of course it has solution for all $y>0$ .






share|cite|improve this answer









$endgroup$



Follow the method in http://en.wikipedia.org/wiki/Method_of_characteristics#Example:



$dfrac{dy}{dt}=1$ , letting $y(0)=0$ , we have $y=t$



$dfrac{du}{dt}=0$ , letting $u(0)=u_0$ , we have $u=u_0$



$dfrac{dx}{dt}=u=u_0$ , letting $x(0)=f(u_0)$ , we have $x=u_0t+f(u_0)=uy+f(u)$ , i.e. $u=F(x-uy)$



$u(x,0)=-x$ :



$F(x)=-x$



$therefore u=-(x-uy)=-x+uy$



Since $u(x,y)$ has perfect explicit form $u(x,y)=dfrac{x}{y-1}$ , of course it has solution for all $y>0$ .







share|cite|improve this answer












share|cite|improve this answer



share|cite|improve this answer










answered Jul 30 '13 at 2:07









doraemonpauldoraemonpaul

12.6k31660




12.6k31660








  • 1




    $begingroup$
    I consider this solution be wrong (partially). The picture higher shows the real situation with characteristics -- they intersect at $y=1$, beginning from that point solution does not exist.
    $endgroup$
    – cool
    Jul 30 '13 at 13:09














  • 1




    $begingroup$
    I consider this solution be wrong (partially). The picture higher shows the real situation with characteristics -- they intersect at $y=1$, beginning from that point solution does not exist.
    $endgroup$
    – cool
    Jul 30 '13 at 13:09








1




1




$begingroup$
I consider this solution be wrong (partially). The picture higher shows the real situation with characteristics -- they intersect at $y=1$, beginning from that point solution does not exist.
$endgroup$
– cool
Jul 30 '13 at 13:09




$begingroup$
I consider this solution be wrong (partially). The picture higher shows the real situation with characteristics -- they intersect at $y=1$, beginning from that point solution does not exist.
$endgroup$
– cool
Jul 30 '13 at 13:09


















draft saved

draft discarded




















































Thanks for contributing an answer to Mathematics Stack Exchange!


  • Please be sure to answer the question. Provide details and share your research!

But avoid



  • Asking for help, clarification, or responding to other answers.

  • Making statements based on opinion; back them up with references or personal experience.


Use MathJax to format equations. MathJax reference.


To learn more, see our tips on writing great answers.




draft saved


draft discarded














StackExchange.ready(
function () {
StackExchange.openid.initPostLogin('.new-post-login', 'https%3a%2f%2fmath.stackexchange.com%2fquestions%2f454776%2fburgers-equation-u-y-uu-x-0-with-ux-0-x%23new-answer', 'question_page');
}
);

Post as a guest















Required, but never shown





















































Required, but never shown














Required, but never shown












Required, but never shown







Required, but never shown

































Required, but never shown














Required, but never shown












Required, but never shown







Required, but never shown







Popular posts from this blog

Plaza Victoria

In PowerPoint, is there a keyboard shortcut for bulleted / numbered list?

How to put 3 figures in Latex with 2 figures side by side and 1 below these side by side images but in...